Understanding Complex Numbers: Limits & Functions

Click For Summary
A function f(z) has a limit l as z approaches z_0 if f is defined near z_0, excluding possibly at z_0 itself. For every positive real ε, there exists a positive real δ such that for all z not equal to z_0 within the disk |z-z_0|<δ, the condition |f(z)-l|<ε holds true. This means that as z gets close to z_0, the values of f(z) approach l, even if f(z_0) is undefined. The discussion confirms that understanding limits involves recognizing how f(z) behaves within the δ disk around z_0. This foundational concept is essential for grasping complex numbers and their functions.
chaoseverlasting
Messages
1,050
Reaction score
3
[SOLVED] Complex Numbers

A very happy new year to all at PF.

Homework Statement


Kreyszig, P.665 section 12.3,

A function f(z) is said to have the limit l as z approaches a point z_0 if f is defined in the neighborhood of z_0 (except perhaps at z_0 itself) and if the values of f are "close" to l for all z "close" to z_0; that is, in precise terms, for every positive real \epsilon we can find a positive real \delta such that for all z not equal to z_0 in the disk |z-z_0|&lt;\delta, we have

|f(z)-l|&lt;\epsilon...(2)

that is, for every z not equal to z_0 in that \delta disk, the value of f lies in the disk (2).

I think this means that if you were to plot z and f(z), for all values of z near the point z_0 (within the \delta disk), but not necessarily at z_0 itself (as the function may not exist at z_0), the value of f(z) would be very close to l (but not necessarily l, as f(z_0) may not exist) and that this value of f(z) would be inside the \epsilon disk.

Is that right?
 
Physics news on Phys.org
sounds good
 
Congrats, you just understood what a limit is.
 
Thank you.
 
Question: A clock's minute hand has length 4 and its hour hand has length 3. What is the distance between the tips at the moment when it is increasing most rapidly?(Putnam Exam Question) Answer: Making assumption that both the hands moves at constant angular velocities, the answer is ## \sqrt{7} .## But don't you think this assumption is somewhat doubtful and wrong?

Similar threads

  • · Replies 1 ·
Replies
1
Views
1K
  • · Replies 22 ·
Replies
22
Views
4K
  • · Replies 14 ·
Replies
14
Views
3K
  • · Replies 3 ·
Replies
3
Views
11K
  • · Replies 2 ·
Replies
2
Views
2K
  • · Replies 7 ·
Replies
7
Views
3K
  • · Replies 2 ·
Replies
2
Views
1K
  • · Replies 8 ·
Replies
8
Views
3K
Replies
4
Views
2K
  • · Replies 6 ·
Replies
6
Views
5K